Cualquiera de las dos rectas puede ser la de pendiente 1 o pendiente 2, sólo hay que
conservarla hasta el final de la operación.
BLO
Fórmula que vamos a aplicar:
m-m
1+m-m
tan 0=
Pendiente de la recta 3x - 4y + 8 = 0
Pendiente de la recta 2x + 3y - 23 = 0
Sustitución de datos
tan 0 =
Al realizar la operación de la fórmula se obtiene la tangente del ángulo, por lo que hay
que buscar el inverso de la tangente, que es el valor del ángulo de intersección entre
las rectas dadas.
tan 8 =
El valor del ángulo e es

Answers

Answer 1
Lo siento, no sé español, quizá alguien más te ayudaría

Related Questions

Simplest form of 8/36

Answers

Answer:

8.36 I think

Step-by-step explanation:

Answer: 2/9

Steps: 8/36 simplified is 2/9.

Plz mark brainliest:)

Show that each conjecture is false by finding a counterexample.


1.For any integer n, n3 > 0.


2. Each angle in a right triangle has a different measure.


3.For many years in the U.S, each bank printed its own currency. The variety of different bills led to widespread counterfeiting. By the time of the Civil War, a significant fraction of the currency in circulation was counterfeit. If one soldier had 48 bills, 16 of which were counterfeit, and another soldier had 39 bills, 13 of which were counterfeit, make a conjecture about what fraction of bills were counterfeit at the time of the Civil War.

Answers

Answer:

1.   [tex]n: n^3 > 0[/tex] is not true for negative integers

2.  [tex]45\ and\ 45[/tex] show that all angles do not have different measure

3.  [tex]Fraction = \frac{1}{3}[/tex]

Step-by-step explanation:

Solving (1):

[tex]n: n^3 > 0[/tex]

Take

[tex]n = -1[/tex]

[tex]-1^3 = -1[/tex]

[tex]n: n^3 > 0[/tex] is not true for negative integers

Solving (2):

The angles of a right angled triangle are:

[tex]45, 45\ and\ 90[/tex]

[tex]45\ and\ 45[/tex] show that all angles do not have different measure

Solving (3):

When

[tex]Bills = 48[/tex]

[tex]Counterfeit = 16[/tex]

The fraction is calculated as thus:

[tex]Fraction = \frac{Counterfeit}{Bills}[/tex]

[tex]Fraction = \frac{16}{48}[/tex]

[tex]Fraction = \frac{1}{3}[/tex]

Similarly, when

[tex]Bills = 39[/tex]

[tex]Counterfeit = 13[/tex]

[tex]Fraction = \frac{Counterfeit}{Bills}[/tex]

[tex]Fraction = \frac{13}{39}[/tex]

[tex]Fraction = \frac{1}{3}[/tex]

Pls I need help ASAP

Answers

Answer:

the answer is 1/8

thank you

if you loved my answer then please like it and mark as brainliest

subtract 8 from the input to get the output
divided the input by 3 to get the output

Answers

Answer: that will be 5

Step-by-step explanation:

Answer:

6

Step-by-step explanation:

f(x) = x + 7, g(x) = x - 7
(a) (f + g)(x) =
(b) (f-g)(x)=
(c) (fg)(x)=
(d) (f/g)(x)=

Answers

a) 2x

b) 14

c) x^2 -49

d) -1

2x+3y=10 8x+Ay=B
What Is the vaule of B

Answers

X-1/8 b-1/8ay hopefully this helps

To win a trivia game you must score 90 points each correct answer is worth 1 3/4 points, each incorrect point is worth -1/4 points if he gets 58 questions correct and 22 questions incorrect, what is his score?

Answers

96 because make 1 3/4 an improper fraction and turn into a decimal it will be 1.75 then multiple by 55(how many you got right) it’ll be 101.5 then 1/4= .25 in decimal form so multiple that by 22(how many you got incorrect) and it’ll be 5.5 so you’ll do 101.5 - 5.5 and it’ll be 96 hope it helps!!

Megan had $235.17 in her bank account. She withdrew $77.98 from the account to spend
on clothing. How much money does she have left in her bank account now?

Answers

Answer: 157.19

Step-by-step explanation:

157.19 that would be the answer

2/15 = h/125 need a step by step please

Answers

Answer:  h = 50/3

====================================

Work Shown:

2/15 = h/125

2*125 = 15*h .... cross multiply

250 = 15h

15h = 250

h = 250/15

h = (5*50)/(5*3)

h = 50/3

-----------------

Extra info:

50/3 = (48+2)/3

50/3 = (48/3)+(2/3)

50/3 = 16 + (2/3)

50/3 = 16 & 2/3

The improper fraction 50/3 converts to the mixed number 16 & 2/3

Another way to see this is to use long division to get 50/3 = 16 remainder 2.

ax+6=15 a is a negative, what must be true about x?

Answers

Answer:

x= 9/a

Step-by-step explanation:

At Sakaro's Sushi, you can get 18 wraps for $30.06 What is the price for 1 wrap?

Answers

Answer: $1.67

Let's make an equation for the problem. To find the price for each sushi roll, divide the price by the number of sushi rolls.

30.06 ÷ 18 = 1.67

Need help ASAP with my math

Answers

U need a calculator for that lol

Answer:  The answer is -2

Step-by-step explanation: I want points and trust me this answer is correct

(-8) X 2/3 i NEED the answer help me

Answers

-5.33333333333

I used a calulator can I get brainliest please

HELP URGENT !!!!!!!!!!!

Answers

The answer is B. -14*1=-14 and -14+1=-13.
I believe the answer is B. Please correct me if I’m wrong! Good luck!

Quadrilateral GHJK has vertices G(2, 3), H(8, 2), J(6, 8), and K(3, 6). It is transformed according to the rule T(–4, –5).

What are the coordinates of G”?

(–7, 3)
(–2, 2)
(–1, –7)
(2, –2)

Answers

Answer:

the transformation T(a, b) can be change the pre image to the final image by following the rule as we explain below

if A(x, y) is a pre image, then A ' ( x', y') is its image, and x' = a+x, y' = b + y, where because of T(a, b) transformation

in our case Quadrilateral GHJK has vertices G(2, 3), so the coordinates of G”

can be found with

T(-4, -5) applied to G(2, 3) ⇒ G' (-4 +2, -5+3)=G' (-2, -2)

T(-4, -5) applied to G' (-2, -2) ⇒ G"(-4-2, -5-2) =G"(-6, -7)

the coordinates of G” are (-6, -7)

Step-by-step explanation:

i hope it helps?

Answer:

its c on edg (-1, -7)

Step-by-step explanation:

11. Factor for

12 – 8.1 + 16​

Answers

I’m going to go ahead and say the answer is 19.9... sorry if it’s wrong.

Answer:

2(6-4.05+8)

Step-by-step explanation:

2(6-48+43.05+8)

Can someone please help me I need help and I can’t understand this plsssss

Answers

1a. Multiply a variable by 13, add 2.

1b. Subtract two from both sides, divide both sides by 13. X=3

2a. Subtract 3 from a variable; divide by 5

2a. Multiply both sides by 5,add 3 to each side. x=-2
Solution (1) :

Solution (a) :

step 1. Multiply the variable x with 13 .

step 2. Add 2 to it .

Solution (b) :

Step 1 : Subtract 2 from both sides : 13x + 2 - 2 = 41 - 2 = 39

Step 2 : Divide both the sides by 13 : 13x ÷ 13 = 39 ÷ 13 = 3

x = 3

Solution (2) :

Solution (a) :

Step 1. Subtract 3 from the variable .

Step 2. Divide the result by 5 .

Solution (b) :

[tex]Step \: 1. \: add\: 3 \: to \: both \: the \: sides : \frac{x - 3}{5} + 3 = - 1 + 3

[/tex]

Step 2 : Multiply both the sides by 5 : 5 × 1/5x = 5 × -2/5

x = -2

Evaluate 8 ÷ -2 · 4²+9. Thanks! Will give brainiest if possible

Answers

the answer is -55 , because you have to simply the 14^2 to 16
For more anwsers for questions like these use math away love even thought the ads might be irritating it’s a great source ✨

The toco toucan, the largest member of the toucan family, possesses the largest beak relative to body size of all birds. This exaggerated feature has received various interpretations, such as being a refined adaptation for feeding. However, the large surface area may also be an important mechanism for radiating heat (and hence cooling the bird) as outdoor temperature increases. Here are data for beak heat loss, as a percent of total body heat loss, at various temperatures in degrees Celsius:

Temperature (degrees Celsius) 15 16 17 18 19 20 21 22 23 24 25 26 27 28 29 30
Percent heat loss from beak 35 36 38 28 41 43 55 46 39 54 45 58 60 56 62 67
a. The equation of the least-squares regression line for predicting beak heat loss, as a percent of total body heat loss from all sources, from temperature is: _____ (Use decimal notation. Give your answer to four decimal places.)

b. Use the equation to predict (
±
0.01) beak heat loss, as a percent of total body heat loss from all sources, at a temperature of 25 degrees Celsius.

c. What percent (
±
0.01) of the variation in beak heat loss is explained by the straight-line relationship with temperature?

d. Find the correlation r (
±
0.001) between beak heat loss and temperature.

Answers

Answer:

Kindly check explanation

Step-by-step explanation:

Given the data:

Temperature (degrees Celsius) : 15 16 17 18 19 20 21 22 23 24 25 26 27 28 29 30

Percent heat loss from beak : 35 36 38 28 41 43 55 46 39 54 45 58 60 56 62 67

Using an online regression calculator ; the regression equation obtained is :

ŷ = 2.0927X + 0.6029

X = independent variable

Y = predicted variable

2.0927 = slope

0.6029 = intercept

B.) temperature = 25

ŷ = 2.0927(25) + 0.6029

= 52.9204

C.) the explained variance is the value of the coefficient of determination (R²) which is the square of the correlation Coefficient

0.8785² = 0.7718

D.) the correlation Coefficient r is 0.8785 using the Coefficient of regression calculator

ayer 100% A company sells cases of tomato juice. Each case contains 24 cans of juice and sells for $18. The equation y = 18x represents the total price ( y), in dollars, when x number of cases are sold. Choose all statements about the situation that are correct based on the information. A. The cost per can is dependent on the number of cases sold. B. The number of cans sold is dependent on the price per can. C. The total price for a sale is dependent on the number of cases sold. D. The number of cases sold is independent from the number of cans in each case.​

Answers

Answer:

c

Step-by-step explanation:

Answer:

C is the answer

3. At a road work site, 20 cones are placed along 50 feet of road. How many cones are placed
along 35 feet of road? *

Answers

Answer:

14 cones

Step-by-step explanation:

20/50=0.4

0.4 times 35=14

The angle measures of triangle QRS are given:

The measure of angle Q is (5x−15)°.
The measure of angle R is 75°.
The measure of angle S is (3x)°.
What is the value of x?

Answers

Answer:

x = 15°

Step-by-step explanation:

(5x-15) + 75 + 3x = 180

8x + 60 = 180

8x = 120

x = 15°

x+12x-13 = 0 and azo, what
is the value x+lo?​

Answers

Wat this doesn’t make sense

The yield in bushels from a grove of orange trees is given by Y = x(600 − x), where x is the number of orange trees per acre. How many trees will maximize the yield?

Answers

Answer:

90000

Step-by-step explanation:

Y = x(600 - x)

Let's rearrange the equation so that we have

Y = x(-x + 600)

Y = -x² + 600

When you take a look at parabolas, we know that their maximum or minimum values are usually on the axis of symmetry. This axis of symmetry has an equation of

x = -b/2a,

In this question, the axis of symmetry is

x = -600 / -2

x = 300

This means that planting 300 trees will maximize your yield. If you substitute for x = 300 into the equation, the maximum yield will be

y = 300 (600 - 300)

y = 90000.

Answer: 300

Step-by-step explanation:

use the formula X=-b/2a

x=-600/-2

x=300

What’s the difference between m=change in y and change in x and m =y2-y1and x2-x1

Answers

it's the same thing

[tex]slope = \frac{Δy}{Δx} or \frac{change \: in \: y}{change \: in \: x} = \frac{y2 - y1}{x2 - x1} = \frac{rise}{run} [/tex]

бx + 8 = 50
I need that answer

Answers

Answer:

=

7

Step-by-step explanation:

Write an equation of the line passing through point P(−2, 6) that is parallel to the line x=−5.

Answers

Answer:

x = -2

Step-by-step explanation:

The line x = - 5 means it is a straight vertical line that has no slope. This means that the slope is undefined.

Now, for this line to be parallel to the line passing through point P(−2, 6), it means the line will be a vertical line too with the equation of form (x = a).

Thus, the equation of the line is x = -2

The equation of the line is given by:

[tex]x = -2[/tex]

The equation of a line has the following format:

[tex]y = mx + b[/tex]

In which:

m is the slope, which is the rate of change.b is the y-intercept, which is the value of x when y = 0.If two lines are parallel, they have the same slope.

Line x = -5 is a horizontal line, thus, it's slope is undefined.

A line parallel will also be horizontal, with undefined slope.

It passes through point P(-2,6), thus, the equation of the line is:

[tex]x = -2[/tex]

A similar problem is given at https://brainly.com/question/16302622

If a person walks 1/3 of a mile every 1/6 of an hour, what is their speed in miles per hour?

Answers

Answer:

2 mi per hour

Step-by-step explanation:

1/3*6=

6/3= 2

2 mi per hour

Answer:

2mph

Step-by-step explanation:

If a person walks 1/3 of a mile every 1/6 of an hour you divide to solve.

[tex]\frac{\frac{1}{3}}{\frac{1}{6}}[/tex] To solve you flip the bottom and multiply:

[tex]\frac{1}{3} *\frac{6}{1}[/tex] = [tex]\frac{6}{3} = 2[/tex]

The person is walking 2mph

9. Determine whether KM and ST are parallel, perpendicular, or neither. K(-1, -8), M(1,6), S(-2,-6), T(2, 10) A Parallel B Perpendicular C Neither​ Hint(use slope formula)

Answers

Neither: the slope for KM=7, and the slope for ST= 1

There are two squares.
The larger square has an area of 1,089 square feet and the smaller square has an area of 121 square inches.
What is the scale factor between the two squares?
(A) 3
B
6
12
D 36

Answers

Answer:

It is 3

Step-by-step explanation:

Answer:

It is 3

Step-by-step explanation:

Other Questions
the image down below. what must animals do for celluar respiraton to begin Which force keeps the planets from floating off into space?O Earth's gravityO the Sun's gravityO the moon's gravityO Milky Way's gravity x2 + 45x = -200 Using the quadratic formual and the discirimnat Write a slope intercept equation for line with the given characteristics m=-3/5 passes through (4,-4) The formula (x)= f(x) 1+f(x)23/2 expresses the curvature of a twice-differentiable plane curve as a function of x. Use this formula to find the curvature function of the following curve. f(x)=5x2 The curvature function is (x)=nothing. .To find a shoe size, the manufacturer uses the equation S = 3f - 22, where Srepresents the shoe size and f represents the heel-to-toe length of the foot, ininches. Which of the following is the same equation solved for fin terms ofS? Which are types of mergers that involve corporations that have created subsidiaries? Check all that apply.defactoshort-formstatuatoryhostile takeoverstriangular Melanie had 3 pounds of dried fruit. She packed the dried fruit into bags of 3/8 pound each. How many bags of dry fruit does she pack? Managers must be able to determine whether their workers are doing an effective and efficient job, with a minimum of errors and disruptions. They do so by using a performance appraisal, an evaluation that measures employee performance against established standards in order to make decisions about promotions, compensation, training, or termination. Managing effectively means getting results through top performance. That's what performance appraisals at all levels of the organization are forincluding at the top, where managers benefit from review by their subordinates. In the 360-degree review, management gathers opinions from all around the employee, including those under, above, and on the same level, to get an accurate, comprehensive idea of the worker's abilities.a. Trueb. False What was one main point of Daltons atomic theory?A. That atoms were made up of positive and negative chargesB. That atoms changed as they formed compoundsC. That atoms has a nucleus at the centerD. That atoms made up the smallest form of matter Help me plz!!!!!!!!!!! 10.The density of aluminum is 2.70 g/mL. If the mass of a piece of aluminum is 244 grams, what is the volume of the aluminum? What is the measure of angle x? 62" X Which expressions are equivalent to -6(b + 2) + 8?Choose all answers that apply:A -66 + 2 + 8B - 66 - 4C None of the above In a system where Round Robin is used for CPU scheduling, the following is TRUE when a process cannot finish its computation during its current time quantum? The process will terminate itself. The process will be terminated by the operating system. The process's state will be changed from running to blocked. None of the mentioned. when was the earliest art museum established during what time period How has the rise of mobile development and devices impacted the IT industry, IT professionals, and software development What is the kinetic Energy of the 80 kg skydiver if he is falling at 60 m/sec? The height of a painting is ft. Between which two consecutive whole numbers is ?Choose the correct answer below.A. and B. and C. and